GitModel-动手学数理统计_03(python)

1 动手学数理统计_03

github 上pdf版本及ipynb版本:https://github.com/cx-333/Math-Modeling

目录

  • 1 动手学数理统计_03
    • 1.9 假设检验之基本思想
    • 1.10 假设检验之正态总体参数的假设检验

1.9 假设检验之基本思想

  • 假设检验问题:在总体的分布函数完全未知或只知其形式、但不知其参数的情况下,为了推断总体的某些未知特性,便提出关于总体的假设,例如,假设某糖果车间生产的糖果的均值为 0.5 k g 0.5kg 0.5kg;需要根据已知的样本(例如某一天生产的糖果)对所提出的假设(判断糖果均值是否为 0.5 k g 0.5kg 0.5kg)作出是接受(糖果均值是 0.5 k g 0.5kg 0.5kg)还是拒绝(糖果均值不是 0.5 k g 0.5kg 0.5kg)的决策。假设检验就是做出这一决策的过程。

  • 引入:某生产葡萄糖车间,袋装糖的质量是一个随机变量,服从正态分布,在车间正常工作时,袋装糖质量的均值为 0.5 k g 0.5kg 0.5kg,标准差为 0.015 k g 0.015kg 0.015kg,某日车间生产了 9 9 9袋葡糖糖,其袋装质量为:
    0.479 0.506 0.518 0.524 0.498 0.511 0.520 0.515 0.512 0.479 \quad 0.506 \quad 0.518 \quad 0.524 \quad 0.498 \quad 0.511 \quad 0.520 \quad 0.515 \quad 0.512 0.4790.5060.5180.5240.4980.5110.5200.5150.512
    问当日机器是否正常工作?

假设标准差是一意已知量,现在要判断当日机器是否正常工作即判断当日所生产的样本的均值是否等于正常工作时的均值即可,于是,提出如下假设
H 0 : μ = 0.5 ; H 1 : μ ≠ 0.5 H_{0}: \mu = 0.5 ; \quad H_{1}:\mu \ne 0.5 H0:μ=0.5;H1:μ=0.5
假设 H 0 H_{0} H0表示当日生产的样本的均值等于正常工作时的均值,即机器正常工作,假设 H 1 H_{1} H1表示当日生产的样本的均值不等于正常工作时的均值,即机器不正常工作。

  • 分析:如何对上述假设做出决策呢?根据引入知,假设是对未知参数做出假设,要通过样本对未知参数做出的假设进行决策,则需要构造一个关于样本和未知参数的函数,上一节区间估计的计算也有和这里相似的一步,没错,就是枢轴量,可以通过枢轴量将样本和未知参数联系起来,不过在假设检验里将枢轴量称为检验统计量,在本例中构造检验统计量(枢轴量)
    X ‾ − μ σ / n \frac{\overline{X} - \mu}{\sigma/\sqrt{n}} σ/n Xμ
    其实,只判断 ∣ x ‾ − μ ∣ |\overline{x} - \mu| xμ的大小也可以做出做出决策,但取上述的检验统计量会使决策的过程更加简便化。

因为,当 H 0 H_{0} H0为真时, X ‾ − μ σ / n ∼ N ( 0 , 1 ) \frac{\overline{X} - \mu}{\sigma/\sqrt{n}} \sim N(0, 1) σ/n XμN(0,1)。而判断 ∣ x ‾ − μ ∣ |\overline{x} - \mu| xμ的大小也可以转化为判断 x ‾ − μ σ / n \frac{\overline{x} - \mu}{\sigma/\sqrt{n}} σ/n xμ的大小。适当选定一正数 k k k,使当观察值 x ‾ \overline{x} x满足 x ‾ − μ σ / n ≥ k \frac{\overline{x} - \mu}{\sigma/\sqrt{n}}\ge k σ/n xμk时就拒绝 H 0 H_{0} H0,反之, x ‾ − μ σ / n < k \frac{\overline{x} - \mu}{\sigma/\sqrt{n}}< k σ/n xμ<k,就接受 H 0 H_{0} H0.

  然而,由于做出决策的依据是总体中的样本,所以 H 0 H_{0} H0为真时也可能做出拒绝 H 0 H_{0} H0的决策(即做出的决策是错误的决策,例如,判断假设为假的,但实际假设是真的),将犯这种错误的概率记为
P { 当 H 0 为 真 时 拒 绝 H 0 } 或 P θ { 拒 绝 H 0 } , θ 为 假 设 的 未 知 参 数 P\{当H_{0}为真时拒绝H_{0}\} \quad 或 P_{\theta}\{拒绝H_{0}\}, \theta 为假设的未知参数 P{H0H0}Pθ{H0}θ
希望将犯这类错误控制在一定范围内,即给出一个较小的数 α ( 0 < α < 1 ) \alpha(0<\alpha<1) α0<α<1,使犯这类错误的概率不超过 α \alpha α,即
P { 当 H 0 为 真 时 拒 绝 H 0 } ≤ α P\{当H_{0}为真时拒绝H_{0}\} \le \alpha P{H0H0}α

  为确定刚才需要选定的正数 k k k,允许犯这类错误的最大概率为 α \alpha α,将上式取等号
P { 当 H 0 为 真 时 拒 绝 H 0 } = α P\{当H_{0}为真时拒绝H_{0}\} = \alpha P{H0H0}=α
则对于例题,即使
P { ∣ X ‾ − μ σ / n ∣ ≥ k } = α P\{\left | \frac{\overline{X} - \mu}{\sigma/\sqrt{n}} \right| \ge k\} = \alpha P{σ/n Xμk}=α
由于 X ‾ − μ σ / n ∼ N ( 0 , 1 ) \frac{\overline{X} - \mu}{\sigma/\sqrt{n}} \sim N(0, 1) σ/n XμN(0,1),所以 k k k z α / 2 z_{\alpha/2} zα/2便于查表。即
∣ X ‾ − μ σ / n ∣ ≥ k 时 \left | \frac{\overline{X} - \mu}{\sigma/\sqrt{n}} \right| \ge k 时 σ/n Xμk
则拒绝假设 H 0 H_{0} H0,若
∣ X ‾ − μ σ / n ∣ < k 时 \left | \frac{\overline{X} - \mu}{\sigma/\sqrt{n}} \right| < k 时 σ/n Xμ<k
则接受假设 H 0 H_{0} H0 α \alpha α称为显著性水平

python代码(检验引入例题)

from scipy.stats import norm 
import numpy as np 

# 显著性水平 alpha = 0.05
# H0: mu = 0.5; H1: mu != 0.5;  
# sigma = 0.015

x = [0.479, 0.506, 0.518, 0.524, 0.498, 0.511, 0.520, 0.515, 0.512]

mu = 0.5
sigma = 0.015 
alpha = 0.05

k = - norm.ppf(loc=0, scale=1, q=alpha/2)
check = np.abs((np.mean(x) - mu)/(sigma/np.sqrt(9)))

if check >= k:
    print("检验统计量{:.2f} >= {:.2f}, 拒绝假设H0, 则mu != 0.5, 机器工作不正常".format(check, k))
else:
    print("检验统计量{:.2f} < {:.2f}, 接受假设H0, 则mu = 0.5, 机器工作正常".format(check, k))
检验统计量1.84 < 1.96, 接受假设H0, 则mu = 0.5, 机器工作正常

  对于上述例题,假设的未知参数 θ \theta θ为等于或不等于某一假设常量,形如这类的假设检验称为双边假设检验。还有一些其它的问题要求假设的未知参数 θ \theta θ大于等于或小于等于某一假设常量,分别将其称为左边检验右边检验

  • 左边检验:即检验假设
    H 0 : θ ≤ θ 0 ; H 1 : θ > θ 0 H_{0}:\theta \le \theta_{0}; \quad H_{1}:\theta > \theta_{0} H0:θθ0;H1:θ>θ0

  • 右边检验:即检验假设
    H 0 : θ ≥ θ 0 ; H 1 : θ < θ 0 H_{0}:\theta \ge \theta_{0}; \quad H_{1}:\theta < \theta_{0} H0:θθ0;H1:θ<θ0

  • 讨论单边检验的拒绝域

(右边检验)设总体 X ∼ N ( μ , σ 2 ) X \sim N(\mu, \sigma^{2}) XN(μ,σ2) μ 未 知 , σ 已 知 \mu {未知}, \sigma {已知} μ,σ X 1 , X 2 , ⋯   , X n X_{1}, X_{2}, \cdots, X_{n} X1,X2,,Xn是来自总体 X X X的样本,给定显著性书平 α \alpha α,求检验问题
H 0 : μ ≤ μ 0 ; H 1 : μ > μ 0 H_{0}:\mu \le \mu_{0}; \quad H_{1}:\mu > \mu_{0} H0:μμ0;H1:μ>μ0
的拒绝域(默认 H 0 H_{0} H0)。

解:
求拒绝域即寻找满足 H 1 H_{1} H1假设的区间,因 H 0 H_{0} H0中的 μ \mu μ都比 H 1 H_{1} H1中的 μ \mu μ要小,当拒绝 H 0 H_{0} H0时,观察值 x ‾ \overline{x} x会向假设 H 1 H_{1} H1靠拢,即观察值 x ‾ \overline{x} x会偏大,因此,拒绝域的形式为
x ‾ ≥ k ( k 为 适 当 正 数 ) \overline{x} \ge k (k为适当正数) xk(k)
接下来即确定正数 k k k
P { H 0 为 真 时 拒 绝 H 0 } = P { X ‾ ≥ k } = P { X ‾ − μ 0 σ / n ≥ k − μ 0 σ / n } ≤ P { X ‾ − μ σ / n ≥ k − μ 0 σ / n } \begin{aligned} P\left\{ H_{0}为真时拒绝H_{0}\right\} &= P\left\{\overline{X} \ge k \right \} \\ & = P\left\{ \frac{\overline{X} - \mu_{0}}{\sigma/\sqrt{n}} \ge \frac{k - \mu_{0}}{\sigma/\sqrt{n}}\right\} \\ &\le P\left\{ \frac{\overline{X} - \mu}{\sigma/\sqrt{n}} \ge \frac{k - \mu_{0}}{\sigma/\sqrt{n}}\right\} \end{aligned} P{H0H0}=P{Xk}=P{σ/n Xμ0σ/n kμ0}P{σ/n Xμσ/n kμ0}
上式不等号成立是由于 μ ≤ μ 0 , X ‾ − μ σ / n ≥ k − μ 0 σ / n \mu \le \mu_{0}, \frac{\overline{X} - \mu}{\sigma/\sqrt{n}} \ge \frac{k - \mu_{0}}{\sigma/\sqrt{n}} μμ0,σ/n Xμσ/n kμ0,事件 { X ‾ − μ 0 σ / n ≥ k − μ 0 σ / n } ⊂ { X ‾ − μ σ / n ≥ k − μ 0 σ / n } \left\{ \frac{\overline{X} - \mu_{0}}{\sigma/\sqrt{n}} \ge \frac{k - \mu_{0}}{\sigma/\sqrt{n}}\right \} \subset \left\{ \frac{\overline{X} - \mu}{\sigma/\sqrt{n}} \ge \frac{k - \mu_{0}}{\sigma/\sqrt{n}}\right \} {σ/n Xμ0σ/n kμ0}{σ/n Xμσ/n kμ0},要控制 P { H 0 为 真 时 拒 绝 H 0 } ≤ α P\{H_{0}为真时拒绝H_{0}\} \le \alpha P{H0H0}α,只需令
P { X ‾ − μ σ / n ≥ k − μ 0 σ / n } = α P\left\{ \frac{\overline{X} - \mu}{\sigma/\sqrt{n}} \ge \frac{k - \mu_{0}}{\sigma/\sqrt{n}}\right\} = \alpha P{σ/n Xμσ/n kμ0}=α
由于 X ‾ − μ σ / n ∼ N ( 0 , 1 ) \frac{\overline{X} - \mu}{\sigma/\sqrt{n}} \sim N(0, 1) σ/n XμN(0,1),则令 k − μ 0 σ / n = z α \frac{k - \mu_{0}}{\sigma/\sqrt{n}} = z_{\alpha} σ/n kμ0=zα分位点。得检验问题得拒绝域为
x ‾ ≥ k = σ / n z α + μ 0 \overline{x} \ge k =\sigma/\sqrt{n}z_{\alpha} + \mu_{0} \\ xk=σ/n zα+μ0

x ‾ − μ 0 σ / n ≥ z α \frac{\overline{x} - \mu_{0}}{\sigma/\sqrt{n}} \ge z_{\alpha} σ/n xμ0zα

(左边检验)设总体 X ∼ N ( μ , σ 2 ) X \sim N(\mu, \sigma^{2}) XN(μ,σ2) μ 未 知 , σ 已 知 \mu {未知}, \sigma {已知} μ,σ X 1 , X 2 , ⋯   , X n X_{1}, X_{2}, \cdots, X_{n} X1,X2,,Xn是来自总体 X X X的样本,给定显著性书平 α \alpha α,求检验问题
H 0 : μ ≥ μ 0 ; H 1 : μ < μ 0 H_{0}:\mu \ge \mu_{0}; \quad H_{1}:\mu < \mu_{0} H0:μμ0;H1:μ<μ0
的拒绝域(默认 H 0 H_{0} H0)。

解:

拒绝 H 0 H_{0} H0时,均值 μ \mu μ H 1 H_{1} H1靠拢,即 μ \mu μ的无偏估计 x ‾ \overline{x} x较小,取一适当正数 k k k H 0 H_{0} H0的拒绝域形式为:
x ‾ ≤ k \overline{x} \le k xk

H 0 H_{0} H0为真时
X ‾ − μ 0 σ / n ∼ N ( 0 , 1 ) \frac{\overline{X}- \mu_{0}}{\sigma/\sqrt{n}} \sim N(0, 1) σ/n Xμ0N(0,1)
确定 k k k的取值
P { H 0 为 真 时 拒 绝 H 0 } = P { x ‾ ≤ k } = P { X ‾ − μ 0 σ / n ≤ k ‾ − μ 0 σ / n } ≤ P { X ‾ − μ σ / n ≤ k ‾ − μ 0 σ / n } , ( H 0 为 真 时 , μ ≥ μ 0 ) \begin{aligned} P\left\{ H_{0}为真时拒绝H_{0}\right\} &= P\left\{\overline{x} \le k\right\} \\ & = P\left\{\frac{\overline{X}- \mu_{0}}{\sigma/\sqrt{n}} \le \frac{\overline{k}- \mu_{0}}{\sigma/\sqrt{n}}\right\} \\ & \le P\left\{\frac{\overline{X}- \mu}{\sigma/\sqrt{n}} \le \frac{\overline{k}- \mu_{0}}{\sigma/\sqrt{n}}\right\}, (H_{0}为真时,\mu \ge \mu_{0}) \end{aligned} P{H0H0}=P{xk}=P{σ/n Xμ0σ/n kμ0}P{σ/n Xμσ/n kμ0},(H0μμ0)


P { X ‾ − μ σ / n ≤ k ‾ − μ 0 σ / n } = α P\left\{\frac{\overline{X}- \mu}{\sigma/\sqrt{n}} \le \frac{\overline{k}- \mu_{0}}{\sigma/\sqrt{n}}\right\} = \alpha P{σ/n Xμσ/n kμ0}=α

k ‾ − μ 0 σ / n = − z α k = − z α σ / n + − μ 0 \begin{aligned} &\frac{\overline{k}- \mu_{0}}{\sigma/\sqrt{n}} = -z_{\alpha} \\ & k = -z_{\alpha}\sigma/\sqrt{n} + - \mu_{0} \end{aligned} σ/n kμ0=zαk=zασ/n +μ0
得拒绝域
x ‾ ≤ k = − z α σ / n + μ 0 x ‾ − μ 0 σ / n ≤ − z α \begin{aligned} &\overline{x} \le k= -z_{\alpha}\sigma/\sqrt{n} + \mu_{0} \\ &\frac{\overline{x}- \mu_{0}}{\sigma/\sqrt{n}} \le -z_{\alpha} \end{aligned} xk=zασ/n +μ0σ/n xμ0zα

讨论:在右边检验中,为什么令 k − μ 0 σ / n = z α \frac{k - \mu_{0}}{\sigma/\sqrt{n}} = z_{\alpha} σ/n kμ0=zα,而在左边检验中令 k − μ 0 σ / n = − z α \frac{k - \mu_{0}}{\sigma/\sqrt{n}} = -z_{\alpha} σ/n kμ0=zα
因为(左边检验) P { X ‾ − μ σ / n ≤ k − μ 0 σ / n } = ∫ − ∞ k − μ 0 σ / n f ( x ) d x , ( f ( x ) 为 x 的 概 率 密 度 函 数 ) P\left\{ \frac{\overline{X} - \mu}{\sigma/\sqrt{n}} \le \frac{k - \mu_{0}}{\sigma/\sqrt{n}}\right\} = \int_{-\infty}^{\frac{k - \mu_{0}}{\sigma/\sqrt{n}}}f(x)dx, \quad (f(x)为x的概率密度函数) P{σ/n Xμσ/n kμ0}=σ/n kμ0f(x)dx,(f(x)x)要使, P { X ‾ − μ σ / n ≤ k − μ 0 σ / n } = α P\left\{ \frac{\overline{X} - \mu}{\sigma/\sqrt{n}} \le \frac{k - \mu_{0}}{\sigma/\sqrt{n}}\right\}=\alpha P{σ/n Xμσ/n kμ0}=α则取 k − μ 0 σ / n = − z α \frac{k - \mu_{0}}{\sigma/\sqrt{n}} = -z_{\alpha} σ/n kμ0=zα
同理(右边检验) P { X ‾ − μ σ / n ≥ k − μ 0 σ / n } = ∫ k − μ 0 σ / n ∞ f ( x ) d x , ( f ( x ) 为 x 的 概 率 密 度 函 数 ) P\left\{ \frac{\overline{X} - \mu}{\sigma/\sqrt{n}} \ge \frac{k - \mu_{0}}{\sigma/\sqrt{n}}\right\} = \int_{\frac{k - \mu_{0}}{\sigma/\sqrt{n}}}^{\infty}f(x)dx, \quad (f(x)为x的概率密度函数) P{σ/n Xμσ/n kμ0}=σ/n kμ0f(x)dx,(f(x)x)要使, P { X ‾ − μ σ / n ≥ k − μ 0 σ / n } = α P\left\{ \frac{\overline{X} - \mu}{\sigma/\sqrt{n}} \ge \frac{k - \mu_{0}}{\sigma/\sqrt{n}}\right\}=\alpha P{σ/n Xμσ/n kμ0}=α则取 k − μ 0 σ / n = z α \frac{k - \mu_{0}}{\sigma/\sqrt{n}} = z_{\alpha} σ/n kμ0=zα

这里只讨论了犯第一类错误得情况(即H_{0}为真时拒绝H_{0}),犯第二类错误(即H_{0}不真时接受H_{0})情况的分析过程与第一类相似,掌握一种即可,这种只限制第一类错误概率,而不考虑第二类错误概率的假设检验称为显著性检验(显著性水平为 α \alpha α

例子:公司购买了 5 5 5批牛奶,现公司怀疑牛奶掺了水,已知天然牛奶的冰点温度服从正太分布,均值为 − 0.545 -0.545 0.545,标准差为 0.008 0.008 0.008,牛奶掺水可视牛奶的冰点温度升高至近似水的冰点温度,测得公司购买的 5 5 5批牛奶的冰点温度均值 x ‾ = − 0.535 \overline{x}=-0.535 x=0.535,问牛奶是否掺了水?取 α = 0.05 \alpha=0.05 α=0.05.

解:

  1. 作出假设
    H 0 : μ ≤ μ 0 = − 0.545 ; H 1 : μ > μ 0 = − 0.545 H_{0}: \mu \le \mu_{0}=-0.545; \quad H_{1}: \mu > \mu_{0}=-0.545 H0:μμ0=0.545;H1:μ>μ0=0.545
    问题即是判断假设 H 0 H_{0} H0是否为真,通过假设可知该问题是右边检验问题

  2. 找拒绝域
    当拒绝 H 0 H_{0} H0时, μ \mu μ大于 μ 0 \mu_{0} μ0,因此,拒绝域的形式为
    x ‾ ≥ k \overline{x} \ge k xk

  3. 确定常数 k k k
    P { H 0 为 真 时 拒 绝 H 0 } = P { x ‾ ≥ k } = P { X ‾ − μ 0 σ / n ≥ k − μ 0 σ / n } ≤ P { X ‾ − μ σ / n ≥ k − μ 0 σ / n } ( μ ≤ μ 0 ) = α \begin{aligned} P\left\{H_{0}为真时拒绝H_{0}\right\} &= P\left\{ \overline{x} \ge k \right\} \\ & = P\{ \frac{\overline{X} - \mu_{0}}{\sigma/\sqrt{n}} \ge \frac{k - \mu_{0}}{\sigma/\sqrt{n}}\} \\ &\le P\{ \frac{\overline{X} - \mu}{\sigma/\sqrt{n}} \ge \frac{k - \mu_{0}}{\sigma/\sqrt{n}}\} \quad (\mu \le \mu_{0})\\ &= \alpha \end{aligned} P{H0H0}=P{xk}=P{σ/n Xμ0σ/n kμ0}P{σ/n Xμσ/n kμ0}(μμ0)=α

  4. k − μ 0 σ / n = z α \frac{k - \mu_{0}}{\sigma/\sqrt{n}}=z_{\alpha} σ/n kμ0=zα,得
    k = σ / n z α + μ 0 k = \sigma/\sqrt{n}z_{\alpha} + \mu_{0} k=σ/n zα+μ0

    x ‾ ≥ σ / n z α + μ 0 x ‾ − μ 0 σ / n ≥ z α \begin{aligned} &\overline{x} \ge \sigma/\sqrt{n}z_{\alpha} + \mu_{0} \\ &\frac{\overline{x} - \mu_{0}}{\sigma/\sqrt{n}} \ge z_{\alpha} \end{aligned} xσ/n zα+μ0σ/n xμ0zα
    时拒绝 H 0 H_{0} H0

python代码(求解例题)

from scipy.stats import norm 
import numpy as np

n = 5
alpha = 0.05 
mu, sigma = -0.545, 0.008
x_mean = -0.535

z_alpha = - norm.ppf(loc=0, scale=1, q=0.05)
k = (sigma/np.sqrt(n))*z_alpha + mu 

if x_mean >= k:
    print("检验统计量{:.4f} >= {:.4f}, 拒绝假设H0, 即牛奶掺了水".format(x_mean, k))
else:
    print("检验统计量{:.4f} < {:.4f}, 接受假设H0, 即牛奶未掺水".format(x_mean, k))
检验统计量-0.5350 >= -0.5391, 拒绝假设H0, 即牛奶掺了水

1.10 假设检验之正态总体参数的假设检验

  • 单个总体 N ( 0 , 1 ) N(0, 1) N(0,1)均值 μ \mu μ的检验

  (1) σ 2 \sigma^{2} σ2已知,关于 μ \mu μ的检验( Z Z Z检验):上面已经讨论过这种情况,这种情况下,检验统计量为 Z = X ‾ − μ 0 σ / n Z = \frac{\overline{X}-\mu_{0}}{\sigma/\sqrt{n}} Z=σ/n Xμ0

  (2) σ 2 \sigma^{2} σ2未知,关于 μ \mu μ的检验( t t t检验):设总体 X ∼ N ( μ , σ 2 ) X\sim N(\mu, \sigma^{2}) XN(μ,σ2) μ , σ 2 \mu, \sigma^{2} μ,σ2未知,求检验问题
H 0 : μ = μ 0 ; H 1 : μ ≠ μ 0 H_{0}: \mu =\mu_{0}; \quad H_{1}:\mu \ne \mu_{0} H0:μ=μ0;H1:μ=μ0
的拒绝域(显著性水平为 α \alpha α)

  :这时,令检验统计量为
X ‾ − μ 0 S / n , ( S 为 样 本 标 准 差 ) \frac{\overline{X}-\mu_{0}}{S/\sqrt{n}} , \quad (S为样本标准差) S/n Xμ0,(S)
根据之前学的定理知, H 0 H_{0} H0为真时:
X ‾ − μ 0 S / n ∼ t ( n − 1 ) \frac{\overline{X}-\mu_{0}}{S/\sqrt{n}} \sim t(n-1) S/n Xμ0t(n1)
选定适当正数 k k k
P { H 0 为 真 时 拒 绝 H 0 } = P { ∣ X ‾ − μ 0 S / n ∣ ≥ k } = α \begin{aligned} P\left\{ H_{0}为真时拒绝H_{0}\right\} &= P\left\{ \left| \frac{\overline{X}-\mu_{0}}{S/\sqrt{n}} \right| \ge k \right\} &= \alpha \end{aligned} P{H0H0}=P{S/n Xμ0k}=α
k = t α / 2 ( n − 1 ) k = t_{\alpha/2}(n-1) k=tα/2(n1)
则拒绝域为
∣ X ‾ − μ 0 S / n ∣ ≥ k = t α / 2 ( n − 1 ) \left| \frac{\overline{X}-\mu_{0}}{S/\sqrt{n}} \right | \ge k = t_{\alpha/2}(n-1) S/n Xμ0k=tα/2(n1)
这类检验方法称为 t t t检验法

例子:某元件寿命 X ( 单 位 : h ) X(单位:h) Xh服从正态分布 N ( μ , σ 2 ) , μ , σ 2 N(\mu, \sigma^{2}),\mu, \sigma^{2} N(μ,σ2)μ,σ2未知,现测得16只元件寿命如下:
159 280 101 212 224 379 179 264 222 362 168 250 149 260 485 170 \begin{aligned} & 159 \quad 280 \quad 101 \quad 212 \quad 224 \quad 379 \quad 179 \quad 264 \\ & 222 \quad 362 \quad 168 \quad 250 \quad 149 \quad 260 \quad 485 \quad 170 \end{aligned} 159280101212224379179264222362168250149260485170
问元件的平均寿命是否大于 225 h 225h 225h(取 α = 0.05 \alpha = 0.05 α=0.05)?

解:

  1. 作出假设
    H 0 : μ ≤ μ 0 = 225 ; H 1 : μ > μ 0 = 225 H_{0}: \mu \le \mu_{0} = 225; \quad H_{1}: \mu > \mu_{0} = 225 H0:μμ0=225;H1:μ>μ0=225
    拒绝 H 0 H_{0} H0时,观察值 x ‾ \overline{x} x较大,取一适当正数 k k k,拒绝域的形式为
    x ‾ ≥ k \overline{x} \ge k xk

  2. 确定正数 k k k
    P { H 0 为 真 时 拒 绝 H 0 } = P { X ‾ ≥ k } = P { X ‾ − μ 0 S / n ≥ k − μ 0 S / n } ≤ P { X ‾ − μ S / n ≥ k − μ 0 S / n } , ( μ ≤ μ 0 ) = α \begin{aligned} P\left\{H_{0}为真时拒绝H_{0}\right\} &= P\left\{\overline{X} \ge k\right\} \\ &= P\left\{\frac{\overline{X} - \mu_{0}}{S/\sqrt{n}} \ge \frac{k- \mu_{0}}{S/\sqrt{n}}\right\} \\ &\le P\left\{\frac{\overline{X} - \mu}{S/\sqrt{n}} \ge \frac{k- \mu_{0}}{S/\sqrt{n}}\right\}, \quad (\mu \le \mu_{0}) \\ &= \alpha \end{aligned} P{H0H0}=P{Xk}=P{S/n Xμ0S/n kμ0}P{S/n XμS/n kμ0},(μμ0)=α
    k − μ 0 S / n = t α ( n − 1 ) \frac{k- \mu_{0}}{S/\sqrt{n}} = t_{\alpha}(n-1) S/n kμ0=tα(n1)

  3. 则拒绝域为
    x ‾ ≥ k = t α ( n − 1 ) S / n + μ 0 \overline{x} \ge k = t_{\alpha}(n-1)S/\sqrt{n} + \mu_{0} xk=tα(n1)S/n +μ0

    x ‾ − μ 0 S / n ≥ t α ( n − 1 ) \frac{\overline{x} - \mu_{0}}{S/\sqrt{n}} \ge t_{\alpha}(n-1) S/n xμ0tα(n1)
    若满足上列不等式时,拒绝 H 0 H_{0} H0,接受 H 1 H_{1} H1,即元件的平均寿命大于 225 h 225h 225h,反之,则接受 H 0 H_{0} H0,拒绝 H 1 H_{1} H1,即元件的平均寿命小于 225 h 225h 225h

python代码(求解上题)

from scipy.stats import t
import numpy as np

# H0: mu <= 225; H1: mu > 225,  mu0 = 225, alpha=0.05
alpha = 0.05
n = 16
mu0 = 225
x = [159, 280, 101, 212, 224, 379, 179, 264, 222, 362, 168, 250, 149, 260, 485, 170]
x_mean = np.mean(x)
S = np.sqrt(np.sum(np.square(x - x_mean))/(n-1))

check = (x_mean - mu0)/(S/np.sqrt(n))
t_alpha = - t.ppf(df=n-1, q=alpha)

if check >= t_alpha:
    print("检验统计量 : {:.4f} >= {:.4f} (t_alpha(n-1)), 拒绝H0, 接受H1, 即元件平均寿命大于 225 h".format(check, t_alpha))
else:
    print("检验统计量 : {:.4f} < {:.4f} (t_alpha(n-1)), 接受H0, 拒绝H1, 即元件平均寿命小于 225 h".format(check, t_alpha))
检验统计量 : 0.6685 < 1.7531 (t_alpha(n-1)), 接受H0, 拒绝H1, 即元件平均寿命小于 225 h
  • 两个正态总体均值差的检验( t t t检验)

X 1 , X 2 , ⋯   , X n 1 X_{1}, X_{2}, \cdots, X_{n1} X1,X2,,Xn1是来自正态总体 N ( μ 1 , σ 1 2 ) N(\mu_{1}, \sigma_{1}^{2}) N(μ1,σ12)的样本, Y 1 , Y 2 , ⋯   , Y n 2 Y_{1}, Y_{2}, \cdots, Y_{n2} Y1,Y2,,Yn2是来自正态总体 N ( μ 2 , σ 2 2 ) N(\mu_{2}, \sigma_{2}^{2}) N(μ2,σ22)的样本,设两样本独立,样本均值和方差分别为 X ‾ , Y ‾ , S 1 2 , S 2 2 \overline{X}, \overline{Y}, S_{1}^{2}, S_{2}^{2} X,Y,S12,S22,求检验问题
H 0 : μ 1 − μ 2 = δ ; H 1 : μ 1 − μ 2 ≠ δ H_{0}:\mu_{1} - \mu_{2} = \delta; \quad H_{1}:\mu_{1} - \mu_{2} \ne \delta H0:μ1μ2=δ;H1:μ1μ2=δ
δ \delta δ为常数,显著性水平取 α \alpha α

  (1) μ 1 , μ 2 \mu_{1}, \mu_{2} μ1,μ2未知, σ 1 2 = σ 2 2 = σ 2 \sigma_{1}^{2} = \sigma_{2}^{2} = \sigma^{2} σ12=σ22=σ2未知,取检验统计量
t = X ‾ − Y ‾ − δ S w 1 n 1 + 1 n 2 , S w 2 = ( n 1 − 1 ) S 1 2 + ( n 2 − 1 ) S 2 2 n 1 + n 2 − 2 , S w = S w 2 t = \frac{\overline{X} - \overline{Y} - \delta}{S_{w}\sqrt{\frac{1}{n1}+\frac{1}{n2}}}, \quad S_{w}^{2} = \frac{(n1-1)S_{1}^{2}+(n2-1)S_{2}^{2}}{n1+n2-2} , \quad S_{w} = \sqrt{S_{w}^{2}} t=Swn11+n21 XYδ,Sw2=n1+n22(n11)S12+(n21)S22,Sw=Sw2
拒绝域为
∣ x ‾ − y ‾ − δ ∣ s w 1 n 1 + 1 n 2 ≥ t α / 2 ( n 1 + n 2 − 2 ) \frac{\left|\overline{x} - \overline{y} - \delta \right |}{s_{w}\sqrt{\frac{1}{n1}+\frac{1}{n2}}} \ge t_{\alpha/2}(n1+n2-2) swn11+n21 xyδtα/2(n1+n22)
  (2) μ 1 , μ 2 \mu_{1}, \mu_{2} μ1,μ2未知, σ 1 2 , σ 2 2 \sigma_{1}^{2}, \sigma_{2}^{2} σ12,σ22已知,取检验统计量
Z = X ‾ − Y ‾ − δ σ 1 2 n 1 + σ 2 2 n 1 Z = \frac{\overline{X} - \overline{Y} - \delta}{\sqrt{\frac{\sigma_{1}^{2}}{n1} + \frac{\sigma_{2}^{2}}{n1}}} Z=n1σ12+n1σ22 XYδ
拒绝域为
∣ x ‾ − y ‾ − δ ∣ σ 1 2 n 1 + σ 2 2 n 1 ≥ z α / 2 \frac{\left|\overline{x} - \overline{y} - \delta\right|}{\sqrt{\frac{\sigma_{1}^{2}}{n1} + \frac{\sigma_{2}^{2}}{n1}}} \ge z_{\alpha/2} n1σ12+n1σ22 xyδzα/2

  • 正态总体方差的假设检验(单个总体的情况)

  设总体 X ∼ N ( μ , σ 2 ) , μ , σ 2 X \sim N(\mu, \sigma^{2}),\mu, \sigma^{2} XN(μ,σ2)μ,σ2均未知, X 1 , X 2 , ⋯   , X n X_{1}, X_{2}, \cdots, X_{n} X1,X2,,Xn,是来自 X X X的样本,要求检验假设
H 0 : σ 2 = σ 0 2 ; H 1 = σ 2 ≠ σ 0 2 H_{0}:\sigma^{2}=\sigma_{0}^{2}; \quad H_{1}=\sigma^{2} \ne \sigma_{0}^{2} H0:σ2=σ02;H1=σ2=σ02
σ 0 2 \sigma_{0}^{2} σ02为已知常数

由于 S 2 S^{2} S2 σ 2 \sigma^{2} σ2的无偏估计,当 H 0 H_{0} H0为真时,观察值 s 2 s^{2} s2 σ 2 \sigma^{2} σ2应该在 1 1 1附近摆动,而不应过分大于或小于 1 1 1,由前面学习的定理知,当 H 0 H_{0} H0为真时
( n − 1 ) S 2 σ 0 2 ∼ χ 2 ( n − 1 ) \frac{(n-1)S^{2}}{\sigma_{0}^{2}} \sim \chi^{2}(n-1) σ02(n1)S2χ2(n1)
取检验统计量
χ 2 = ( n − 1 ) S 2 σ 0 2 \chi^{2} = \frac{(n-1)S^{2}}{\sigma_{0}^{2}} χ2=σ02(n1)S2
如上所说,上述问题的拒绝域具有以下的形式:
( n − 1 ) S 2 σ 0 2 ≤ k 1 或 ( n − 1 ) S 2 σ 0 2 ≥ k 2 \frac{(n-1)S^{2}}{\sigma_{0}^{2}} \le k_{1} \quad 或 \frac{(n-1)S^{2}}{\sigma_{0}^{2}} \ge k_{2} σ02(n1)S2k1σ02(n1)S2k2
其中 k 1 , k 2 k_{1}, k_{2} k1,k2的值由下式确定
P { H 0 为 真 时 拒 绝 H 0 } P { ( ( n − 1 ) S 2 σ 0 2 ≤ k 1 ) ∪ ( ( n − 1 ) S 2 σ 0 2 ≥ k 2 ) } = α \begin{aligned} &P\left\{H_{0}为真时拒绝H_{0}\right\} \\ &P\left\{(\frac{(n-1)S^{2}}{\sigma_{0}^{2}} \le k_{1}) \cup (\frac{(n-1)S^{2}}{\sigma_{0}^{2}} \ge k_{2}) \right\} & = \alpha \end{aligned} P{H0H0}P{(σ02(n1)S2k1)(σ02(n1)S2k2)}=α
为方便计算,取
P { ( n − 1 ) S 2 σ 0 2 ≤ k 1 } = α 2 P { ( n − 1 ) S 2 σ 0 2 ≥ k 2 } = α 2 P\left\{\frac{(n-1)S^{2}}{\sigma_{0}^{2}} \le k_{1}\right\} =\frac{\alpha}{2} P\left\{ \frac{(n-1)S^{2}}{\sigma_{0}^{2}} \ge k_{2}\right\}=\frac{\alpha}{2} P{σ02(n1)S2k1}=2αP{σ02(n1)S2k2}=2α
k 1 = χ 1 − α / 2 2 ( n − 1 ) , k 2 = χ α / 2 2 ( n − 1 ) ( 这 里 因 为 χ 2 分 布 的 图 像 不 是 对 称 的 ) k_{1}=\chi_{1-\alpha/2}^{2}(n-1), k_{2}=\chi_{\alpha/2}^{2}(n-1)\quad (这里因为\chi^{2}分布的图像不是对称的) k1=χ1α/22(n1),k2=χα/22(n1)(χ2) ,于是得拒绝域为
( n − 1 ) S 2 σ 0 2 ≤ χ 1 − α / 2 2 ( n − 1 ) 或 ( n − 1 ) S 2 σ 0 2 ≥ χ α / 2 2 ( n − 1 ) \frac{(n-1)S^{2}}{\sigma_{0}^{2}} \le \chi_{1-\alpha/2}^{2}(n-1) {或} \frac{(n-1)S^{2}}{\sigma_{0}^{2}} \ge \chi_{\alpha/2}^{2}(n-1) σ02(n1)S2χ1α/22(n1)σ02(n1)S2χα/22(n1)

表1.10 正态总体均值、方差的检验法(显著性水平为 α \alpha α

原假设 H 0 H_{0} H0 检验统计量 备择假设 H 1 H_{1} H1 拒绝域
1 1 1 μ ⩽ μ 0 μ ⩾ μ 0 μ = μ 0 ( σ 2 已 知 ) \begin{aligned} &\mu \leqslant \mu_{0} \\ &\mu \geqslant \mu_{0} \\ &\mu=\mu_{0}\\ &\left(\sigma^{2}\right. 已知)\end{aligned} μμ0μμ0μ=μ0(σ2) Z = X ˉ − μ 0 σ / n Z=\frac{\bar{X}-\mu_{0}}{\sigma / \sqrt{n}} Z=σ/n Xˉμ0 μ > μ 0 μ < μ 0 μ ≠ μ 0 \begin{aligned} &\mu>\mu_{0} \\ & \mu<\mu_{0} \\ & \mu\ne\mu_{0}\end{aligned} μ>μ0μ<μ0μ=μ0 z ≥ z α z ≤ − z α ∣ z ∣ ≥ z α / 2 \begin{aligned} &z\ge z_{\alpha} \\ &z \le -z_{\alpha} \\ & |z| \ge z_{\alpha / 2}\end{aligned} zzαzzαzzα/2
2 2 2 μ ⩽ μ 0 μ ⩾ μ 0 μ = μ 0 ( σ 2 未 知 ) \begin{aligned}&\mu \leqslant \mu_{0} \\ &\mu \geqslant \mu_{0} \\ &\mu=\mu_{0}\\ &\left(\sigma^{2}\right. 未知)\end{aligned} μμ0μμ0μ=μ0(σ2) t = X ‾ − μ 0 S / n \begin{aligned} t =\frac{\overline{X}-\mu_{0}}{S/\sqrt{n}} \end{aligned} t=S/n Xμ0 μ > μ 0 μ < μ 0 μ ≠ μ 0 \begin{aligned} &\mu>\mu_{0} \\ & \mu<\mu_{0} \\ & \mu\ne\mu_{0}\end{aligned} μ>μ0μ<μ0μ=μ0 t ⩾ t α ( n − 1 ) t ⩽ − t α ( n − 1 ) ∣ t ∣ ⩾ t α / 2 ( n − 1 ) \begin{aligned}& t \geqslant t_{\alpha}(n-1) \\&t \leqslant-t_{\alpha}(n-1) \\& |t| \geqslant t_{\alpha / 2}(n-1) \\ \end{aligned} ttα(n1)ttα(n1)ttα/2(n1)
3 3 3 μ 1 − μ 2 ≤ δ μ 1 − μ 2 ≥ δ μ 1 − μ 2 = δ ( σ 1 2 , σ 2 2 已 知 ) \begin{aligned} &\mu_{1}-\mu_{2}\le \delta \\&\mu_{1}-\mu_{2}\ge \delta \\&\mu_{1}-\mu_{2}= \delta\\&(\sigma_{1}^{2},\sigma_{2}^{2}已知) \end{aligned} μ1μ2δμ1μ2δμ1μ2=δ(σ12,σ22) Z = X ‾ − Y ‾ − δ σ 1 2 n 1 + σ 1 2 n 2 \begin{aligned}Z=\frac{\overline{X}-\overline{Y}-\delta}{\sqrt{\frac{\sigma_{1}^{2}}{n1}+\frac{\sigma_{1}^{2}}{n2}}} \end{aligned} Z=n1σ12+n2σ12 XYδ μ 1 − μ 2 > δ μ 1 − μ 2 < δ μ 1 − μ 2 ≠ δ \begin{aligned}&\mu_{1}-\mu_{2}> \delta \\&\mu_{1}-\mu_{2}< \delta \\&\mu_{1}-\mu_{2}\ne \delta \end{aligned} μ1μ2>δμ1μ2<δμ1μ2=δ z ≥ z α z ≤ − z α ∣ z ∣ ≥ z α / 2 \begin{aligned} &z\ge z_{\alpha} \\ & z\le -z_{\alpha}\\ & | z| \ge z_{\alpha/2}\end{aligned} zzαzzαzzα/2
4 4 4 μ 1 − μ 2 ≤ δ μ 1 − μ 2 ≥ δ μ 1 − μ 2 = δ ( σ 1 2 = σ 2 2 = σ 2 未 知 ) \begin{aligned}&\mu_{1}-\mu_{2}\le \delta \\&\mu_{1}-\mu_{2}\ge \delta \\&\mu_{1}-\mu_{2}= \delta\\&(\sigma_{1}^{2}=\sigma_{2}^{2}=\sigma^{2}未知) \end{aligned} μ1μ2δμ1μ2δμ1μ2=δ(σ12=σ22=σ2) t = X ‾ − Y ‾ − δ S w 1 n 1 + 1 n 2 S w 2 = ( n 1 − 1 ) S 1 2 + ( n 2 − 1 ) S 2 2 n 1 + n 2 − 2 \begin{aligned}&t=\frac{\overline{X}-\overline{Y}-\delta}{S_{w}\sqrt{\frac{1}{n1}+\frac{1}{n2}}} \\ & S_{w}^{2}=\frac{(n1-1)S_{1}^{2}+(n2-1)S_{2}^{2}}{n1+n2-2} \end{aligned} t=Swn11+n21 XYδSw2=n1+n22(n11)S12+(n21)S22 μ 1 − μ 2 > δ μ 1 − μ 2 < δ μ 1 − μ 2 ≠ δ \begin{aligned}&\mu_{1}-\mu_{2}> \delta \\&\mu_{1}-\mu_{2}< \delta \\&\mu_{1}-\mu_{2}\ne \delta \end{aligned} μ1μ2>δμ1μ2<δμ1μ2=δ t ≥ t α ( n 1 + n 2 − 2 ) t ≤ − t α ( n 1 + n 2 − 2 ) ∣ t ∣ ≥ t α / 2 ( n 1 + n 2 − 2 ) \begin{aligned}&t\ge t_{\alpha}(n1+n2-2) \\&t \le -t_{\alpha}(n1+n2-2)\\&|t| \ge t_{\alpha/2}(n1+n2-2) \end{aligned} ttα(n1+n22)ttα(n1+n22)ttα/2(n1+n22)
5 5 5 σ 1 2 ≤ σ 2 2 σ 1 2 ≥ σ 2 2 σ 1 2 = σ 2 2 ( μ 未 知 ) \begin{aligned}&\sigma_{1}^{2}\le \sigma_{2}^{2} \\&\sigma_{1}^{2}\ge \sigma_{2}^{2}\\ &\sigma_{1}^{2}= \sigma_{2}^{2}\\ &(\mu 未知) \end{aligned} σ12σ22σ12σ22σ12=σ22(μ) χ 2 = ( n − 1 ) S 2 σ 0 2 \begin{aligned}\chi^{2}=\frac{(n-1)S^{2}}{\sigma_{0}^{2}} \end{aligned} χ2=σ02(n1)S2 σ 1 2 > σ 2 2 σ 1 2 < σ 2 2 σ 1 2 ≠ σ 2 2 \begin{aligned}&\sigma_{1}^{2}>\sigma_{2}^{2} \\&\sigma_{1}^{2}< \sigma_{2}^{2}\\ &\sigma_{1}^{2}\ne \sigma_{2}^{2} \end{aligned} σ12>σ22σ12<σ22σ12=σ22 χ 2 ≥ χ α 2 ( n − 1 ) χ 2 ≤ χ 1 − α 2 ( n − 1 ) χ 2 ≥ χ α / 2 2 ( n − 1 ) 或 χ 2 ≤ χ 1 − α / 2 2 ( n − 1 ) \begin{aligned}&\chi^{2}\ge \chi_{\alpha}^{2}(n-1) \\&\chi^{2}\le \chi_{1-\alpha}^{2}(n-1) \\&\chi^{2}\ge \chi_{\alpha/2}^{2}(n-1)或\\&\chi^{2}\le \chi_{1-\alpha/2}^{2}(n-1) \end{aligned} χ2χα2(n1)χ2χ1α2(n1)χ2χα/22(n1)χ2χ1α/22(n1)
6 6 6 σ 1 2 ≤ σ 2 2 σ 1 2 ≥ σ 2 2 σ 1 2 = σ 2 2 ( μ 1 , μ 2 未 知 ) \begin{aligned}&\sigma_{1}^{2}\le \sigma_{2}^{2} \\&\sigma_{1}^{2}\ge \sigma_{2}^{2}\\ &\sigma_{1}^{2}= \sigma_{2}^{2}\\ &(\mu_{1},\mu_{2}未知)\end{aligned} σ12σ22σ12σ22σ12=σ22(μ1,μ2) F = S 1 2 S 2 2 \begin{aligned}F=\frac{S_{1}^{2}}{S_{2}^{2}} \end{aligned} F=S22S12 σ 1 2 > σ 2 2 σ 1 2 < σ 2 2 σ 1 2 ≠ σ 2 2 \begin{aligned}&\sigma_{1}^{2}>\sigma_{2}^{2} \\&\sigma_{1}^{2}< \sigma_{2}^{2}\\ &\sigma_{1}^{2}\ne \sigma_{2}^{2} \end{aligned} σ12>σ22σ12<σ22σ12=σ22 F ≥ F α ( n 1 − 1 , n 2 − 1 ) F ≤ F 1 − α ( n 1 − 1 , n 2 − 1 ) F ≥ F α / 2 ( n 1 − 1 , n 2 − 1 ) 或 F ≤ F 1 − α / 2 ( n 1 − 1 , n 2 − 1 ) \begin{aligned}&F\ge F_{\alpha}(n1-1, n2-1) \\&F\le F_{1-\alpha}(n1-1, n2-1) \\ & F\ge F_{\alpha/2}(n1-1, n2-1)或\\&F\le F_{1-\alpha/2}(n1-1, n2-1)\end{aligned} FFα(n11,n21)FF1α(n11,n21)FFα/2(n11,n21)FF1α/2(n11,n21)
7 7 7 μ D ≤ 0 μ D ≥ 0 μ D = 0 ( 成 对 数 据 ) \begin{aligned}&\mu_{D}\le 0\\&\mu_{D}\ge 0 \\&\mu_{D}= 0\\&(成对数据)\end{aligned} μD0μD0μD=0() t = D ‾ − 0 S D / n \begin{aligned}t=\frac{\overline{D}-0}{S_{D}/\sqrt{n}} \end{aligned} t=SD/n D0 μ D > 0 μ D < 0 μ D ≠ 0 \begin{aligned}&\mu_{D}> 0\\&\mu_{D}< 0 \\&\mu_{D}\ne 0 \end{aligned} μD>0μD<0μD=0 t ≥ t α ( n − 1 ) t ≤ − t α ( n − 1 ) ∣ t ∣ ≥ t α / 2 ( n − 1 ) \begin{aligned}&t\ge t_{\alpha}(n-1) \\&t\le -t_{\alpha}(n-1)\\&|t|\ge t_{\alpha/2}(n-1) \end{aligned} ttα(n1)ttα(n1)ttα/2(n1)

置信区间与假设检验之间的关系, 假设置信区间 ( θ ‾ , θ ‾ ) (\underline{\theta}, \overline{\theta}) (θ,θ),显著性水平为 α \alpha α

    1. 检验问题 H 0 : θ = θ 0 ; H 0 : θ ≠ θ 0 H_{0}: \theta=\theta_{0};\quad H_{0}: \theta\ne \theta_{0} H0:θ=θ0;H0:θ=θ0 若检验统计量在区间 ( θ ‾ , θ ‾ ) (\underline{\theta}, \overline{\theta}) (θ,θ)内,则接受 H 0 H_{0} H0,否则,拒绝 H 0 H_{0} H0
    1. 检验问题 H 0 : θ ≥ θ 0 ; H 0 : θ < θ 0 H_{0}: \theta\ge \theta_{0};\quad H_{0}: \theta < \theta_{0} H0:θθ0;H0:θ<θ0 若检验统计量在区间 ( − ∞ , θ ‾ ) (-\infty, \overline{\theta}) (,θ)内,则接受 H 0 H_{0} H0

你可能感兴趣的:(数学建模,python,概率论,开发语言)